Difference between revisions of "2025 AMC 8 Problems"

m (Disclaimer)
(Replaced content with "The 2025 AMC 8 is not held yet. '''Please do not post false problems.'''")
(Tag: Replaced)
 
(19 intermediate revisions by 9 users not shown)
Line 1: Line 1:
==Problem 1==
 
 
Let <math>m</math> and <math>n</math> be <math>2</math> integers such that <math>m>n</math>. Suppose <math>m+n=20</math>, <math>m^2+n^2=328</math>, find <math>m^2-n^2</math>.
 
 
<math>\textbf{(A)}\ 280 \qquad \textbf{(B)}\ 292 \qquad \textbf{(C)}\ 300 \qquad \textbf{(D)}\ 320 \qquad \textbf{(E)}\ 340</math>
 
 
==Problem 2==
 
 
 
The 2025 AMC 8 is not held yet. '''Please do not post false problems.'''
 
The 2025 AMC 8 is not held yet. '''Please do not post false problems.'''

Latest revision as of 10:28, 31 July 2024

The 2025 AMC 8 is not held yet. Please do not post false problems.